You are on page 1of 3

Homework # 5

10.6. (a).
 X 2
n o 1 1
P min{X, Y } > i = P {X > i}P {Y > i} = k
= i
2 4
k=i+1

Thus n o 1
P min{X, Y } i = 1 i
4
(b).

X X X 1 1 1 1
P {X = Y } = P {X = i, Y = i} = P {X = i}P {Y = i} = i
= =
i=1 i=1 i=1
4 414 1 3

(c). Notice that

P {X > Y } + P {Y > X} + P {X = Y } = 1 andP {X > Y } = P {Y > X}

Thus
1 o 1 1 1
P {Y > X} = 1 P {X = Y } = 1 =
2 2 3 3
(d).
X
n X [  X 1 1
P X devides Y } = P {X = i, Y = ij} =
i=1 j=1 i=1 j=1
2i 2ij

X 1 1 X 1 1
= = 2
i=1
2i(1+i) 1 2i i=1
2i 2i 1

(e)
X

X X 1 1 X 1 1
P {X kY } = P {X ki, Y = i} = j i
=2
2 2 2i 2ki
i=1 i=1 j=ki i=1
2 1 2
= =
21+k 1 2(1+k) 21+k 1

10.12. [

\  [ 
P {An i.o.} = P Ak = lim P Ak
n
n=1 k=n k=n

Notice that

[ 
P Ak P (An )
k=n

1
Taking limsup on the both side (notice the limit exists on the left hand side by monotonic-
ity),
[ 
lim P Ak lim sup P (An )
n n
k=n

10.13. Assume complete convergence. By the first part of Borel-Cantelli lemma (the
part does not need independence).
[
\ o

P |Xn X| > i.o. = P {|Xk X| > } = 0
n=1 k=n

Or, equivalently, 
P |Xn X| eventually = 1
Thus,
lim sup |Xn X| a.s.
n

Notice that > 0 can be arbitrarily small, letting 0+ on the right leads to

lim Xn = X a.s. ()
n

On the other hand, assume (*) holds. By 0-1 law, X is equal to a constant almost
surely. Therefore, the sequence {Xn X} is independent.
For any > 0,
[
\ o n o

P {|Xk X| > } = P |Xn X| > i.o. P lim Xn 6= X = 0
n
n=1 k=n

By the second part of Borel-Cantelli lemma (Here the independence is needed), we must
have
X
P {|Xn X| > } <
n=1

10. 16* First, A1 , , Ak , are independent. By Borel-Cantelli lemma. All we need


to show is that
X = if p 1/2
P (Ak )
< if p < 1/2

k=1

Fix k and write

Tk =The starting time of the first consecutive head-run during


[2k , 2k+1 1] that last at least k rounds

2
Then
2k+1
Xk
P (Ak ) = P {2k Tk 2k+1 k} = P {Tk = j}
j=2k

For j > 2k , P {Tk = j} = (1 p)pk . For j = 2k , P {Tk = j} = pk . Thus

P (Ak ) = pk + (1 p)(2k 2)pk (2p)k (k )

Therefore, the conclusion follows from the fact that



X
(2p)k <
k=1

if and only if p < 1/2.

10.18. By the relation X = a Y a.s. and by Theorem 10.1-(c), X is independent of


itself. Consequently, for any number x,

P {X x} = P {X x, X x} = P {X x}2

Therefore, P {X x} = 0 or 1. Notice that the distribution function F (x) = P {X x}


is non-decreasing. There is a C such that F (x) = 0 as x < C and F (x) = 1 as x > C.
Hence, X = C a.s. Thus, Y = a C a.s.

You might also like